2020 AMC 12A Problems/Problem 24

Revision as of 14:06, 16 March 2020 by Siluweston (talk | contribs) (Solution 3 (Answer Choices))

Problem 24

Suppose that $\triangle{ABC}$ is an equilateral triangle of side length $s$, with the property that there is a unique point $P$ inside the triangle such that $AP=1$, $BP=\sqrt{3}$, and $CP=2$. What is $s$?

$\textbf{(A) } 1+\sqrt{2} \qquad \textbf{(B) } \sqrt{7} \qquad \textbf{(C) } \frac{8}{3} \qquad \textbf{(D) } \sqrt{5+\sqrt{5}} \qquad \textbf{(E) } 2\sqrt{2}$

Solution 1

[asy] draw((0,0)--(4,5.65)--(8,0)--cycle); label("A", (4,5.65), N, p = fontsize(10pt)); label("C", (8,0), SE, p = fontsize(10pt)); label("B", (0,0), SW, p = fontsize(10pt)); label("P", (3.5,3.5), NW, p = fontsize(10pt)); draw((0,0)--(3.5,3.5)); label("$\sqrt{3}$",(0,0)--(3.5,3.5), SE); draw((8,0)--(3.5,3.5)); label("2",(8,0)--(3.5,3.5), SW); draw((4,5.65)--(3.5,3.5)); label("1",(4,5.65)--(3.5,3.5), E); draw((8,0)--(4,5.65)--(11,5.65)--cycle); label("$P'$", (6,4.75), NE, p = fontsize(10pt)); draw((4,5.65)--(6,4.75)); label("1",(4,5.65)--(6,4.75), S); draw((8,0)--(6,4.75)); label("$\sqrt{3}$",(8,0)--(6,4.75), E); draw((3.5,3.5)--(6,4.75)); label("1", (3.5,3.5)--(6,4.75), SE);  [/asy]

We begin by rotating $\triangle{ABC}$ by $60^{\circ}$ about $A$, such that in $\triangle{A'B'C'}$, $B' = C$. We see that $\triangle{APP'}$ is equilateral with side length $1$, meaning that $\angle APP' = 60^{\circ}$. We also see that $\triangle{CPP'}$ is a $30-60-90$ right triangle, meaning that $\angle CPP'= 60^{\circ}$. Thus, by adding the two together, we see that $\angle APC = 120^{\circ}$. We can now use the law of cosines as following:

\[s^2 = (AP)^2 + (CP)^2 - 2(AP)(CP)\cos{\angle{APC}}\] \[s^2 = 1 + 4 - 2(1)(2)\cos{120^{\circ}}\] \[s^2 = 5 - 4(-\frac{1}{2})\] \[s = \sqrt{5 + 2}\]

giving us that $s = \boxed{\textbf{(B) } \sqrt{7}}$. ~ciceronii

Solution 2 (Intuition)

[asy]  unitsize(1inch);  dot((0.756,0.655)); dot((1.512,1.309)); dot((1.701,0.327));  pair A = origin, B = (1.323,2.291), C = (2.646,0), P = (0.756,0.655), Q = (1.512,1.309), R = (1.701,0.327); draw((0,0)--(1.323,2.291)--(2.646,0)--cycle); label("A", (0,0), SW, p = fontsize(10pt)); label("C", (2.646,0), SE, p = fontsize(10pt)); label("B", (1.323,2.291), N, p = fontsize(10pt)); label("P'", (0.756,0.655), NW, p = fontsize(10pt)); label("1", (2.174,0.164), N, p = fontsize(10pt)); label("1", (1.228,0.491), N, p = fontsize(10pt)); D(A--P, red); D(B--Q, red); D(C--R, red); D(P--Q--R--cycle, blue); D(B--P, magenta);  [/asy]

Suppose that triangle $ABC$ had three segments of length $2$, emanating from each of its vertices, making equal angles with each of its sides, and going into its interior. Suppose each of these segments intersected the segment clockwise to it precisely at its other endpoint and inside $ABC$ (as pictured in the diagram above). Clearly $s > 2$ and the triangle defined by these intersection points will be equilateral (pictured by the blue segments).

Take this equilateral triangle to have side length $1$. The portions of each segment outside this triangle (in red) have length $1$. Take $P'$ to be the intersection of the segments emanating from $A$ and $C$. By Law of Cosines, \[BP' = \sqrt{1 + 1 - 2\cos{120^\circ}} = \sqrt{3}.\] So, $P'$ actually satisfies the conditions of the problem, and we can obtain again by Law of Cosines \[s = \sqrt{4 + 1 - 4\cos{120^\circ}} = \boxed{\textbf{(B)} \sqrt{7}}.\]

~ hnkevin42

Solution 3 (Answer Choices)

[asy] draw((0,0)--(4,5.65)--(8,0)--cycle); label("A", (4,5.65), N, p = fontsize(10pt)); label("C", (8,0), SE, p = fontsize(10pt)); label("B", (0,0), SW, p = fontsize(10pt)); label("P", (3.5,3.5), E, p = fontsize(10pt)); label("E", (2.8191,3.982), NW, p = fontsize(10pt)); label("F", (4.848,4.452), NE, p = fontsize(10pt)); label("G", (3.5,0), NE, p = fontsize(10pt)); draw((0,0)--(3.5,3.5)); label("$\sqrt{3}$",(0,0)--(3.5,3.5), SE); draw((8,0)--(3.5,3.5)); label("2",(8,0)--(3.5,3.5), SW); draw((4,5.65)--(3.5,3.5)); label("1",(4,5.65)--(3.5,3.5), E); draw((3.5,3.5)--(2.8191,3.982)); draw((3.5,3.5)--(4.848,4.452)); draw((3.5,3.5)--(3.5,0));  [/asy]

We begin by dropping altitudes from point $P$ down to all three sides of the triangle as shown above. We can therefore make equations regarding the areas of triangles $\triangle{APC}$, $\triangle{APB}$, and $\triangle{BPC}$. Let $s$ be the side of the equilateral triangle, we use the Heron's formula:

\[\triangle{APC} = \frac{s\cdot PF}{2} = \sqrt{\frac{s+3}{2}(\frac{s+3}{2}-s)(\frac{s+3}{2}-1)(\frac{s+3}{2}-2)}\] \[\implies PF = \frac{\sqrt{10s^2-4s-9}}{2s}\]

Similarly, we obtain:

\[PE = \frac{\sqrt{8s^2-s^4-4}}{2s}\] \[PG = \frac{\sqrt{14s^2-s^4-1}}{2s}\]

By Viviani's theorem, \[\frac{\sqrt{10s^2-s^4-9}}{2s}+\frac{\sqrt{8s^2-s^4-4}}{2s}+\frac{\sqrt{14s^2-s^4-1}}{2s} = \frac{\sqrt{3}}{2}s\] \[\sqrt{10s^2-s^4-9}+\sqrt{8s^2-s^4-4}+\sqrt{14s^2-s^4-1} = \sqrt{3}s^2\]

Note that from now on, the algebra will get extremely ugly and almost impossible to do by hand within the time frame. However, we do see that it's extremely easy to check the answer choices with the equation in this form. Testing $\sqrt{7}$, We obtain $7\sqrt{3}$ on both sides, revealing that our answer is in fact $\boxed{\textbf{(B) } \sqrt{7}}$ ~ siluweston

Video Solution

https://www.youtube.com/watch?v=mUW4zcrRL54

See Also

2020 AMC 12A (ProblemsAnswer KeyResources)
Preceded by
Problem 23
Followed by
Problem 25
1 2 3 4 5 6 7 8 9 10 11 12 13 14 15 16 17 18 19 20 21 22 23 24 25
All AMC 12 Problems and Solutions

The problems on this page are copyrighted by the Mathematical Association of America's American Mathematics Competitions. AMC logo.png